Halp meh pls... Which statement verifies that f(x) and g(x) are inverses of each other?
f(g(x))=x
f(g(x))=x and g(f(x))= -x
f(g(x))=x = 1/f(g(x))
f(g(x))=x and g(f(x))=x

Answers

Answer 1

The function pairs are inverses as f(x) = 2x-3 and g(x)=(x+3)/2. The correct answer would be an option (D).

What is an inverse function?

The inverse function is defined as a function obtained by reversing the given function. To determine the inverse of a function, all you have to do is switch where x and y are and resolve for y.

As per the options, we have

f(g(x))=x is expressing f is the inverse of g.

f(g(x))=x is expressing f is the inverse of g but g is not the inverse of f, g(f(x))≠x.

f(g(x))= g(f(x)) is expressing the composition of functions f(g(x)) is equal to the composition of functions g(f(x)), but not necessarily inverses. It doesn't define each equal x.

f(g(x))=x and g(f(x)) =x is expressing f is the inverse of g and g is the inverse of f.

Thus, the function pairs are inverses as f(x) = 2x-3 and g(x)=(x+3)/2.

Hence, the correct answer would be an option (D).

Learn more about inverse functions here:

brainly.com/question/2541698

#SPJ2


Related Questions

PLEASE HELP I WILL GIVE YOU 13 POINTS

Answers

Answer:

I think it would be C.

Step-by-step explanation:

For y [tex]\geq[/tex] the line should be solid and it should be shaded above the line because ≥ means greater than.

For y > the line should be dashed and it should be shaded above the line because it means greater than.

For y [tex]\leq[/tex] the line should be solid and it should be shaded below the line because it means less than.

For y < the line should be solid and it should be shaded below the line because it means less than.

Hope this helps!

y=3x-5 identify y intercept within equation

Answers

Answer: -5

Step-by-step explanation: alrite so when u graph it, the line crosses -5 on the y axis

Answer: -5

Step-by-step explanation:

When written in slope intercept form, which is y=Mx+b, b is the y intercept so in this case it would be -5

What is the equation of the line that passes throligh the point (-6, 6) and has a
slope of -5/3

Answers

Answer:

y = -1 2/3x - 4

Step-by-step explanation:

y = mx + b

Replace numbers with the corresponding variables

6 = -5/3 * -6 + b

6 = 10 + b

-4 = b

y = -1 2/3x - 4

Let me know if something's wrong :)

Write each equation in slope intercept form the graph

Answers

I believe the answer is 4/5x +2. Sorry if I’m wrong.

Triangle ABC will be dilated according to the rule

[tex]D_{F, 0.25} (x,y)[/tex], where point F is the center of dilation.

What will be the coordinates of vertex A' of the image?


a. (-8,-4)

b. (-2,-1)

c. (0, 0)

d. (1, 0)


The answer is (1,0), but could someone explain how you get it, please?

Answers

Usually dilations have a center of dilation at the origin (0,0). When this occurs all we need to do is multiply each coordinate by the dilation scale. Unfortunately things get a little more complicated when the center of dilation is not at the origin.

In this case the center of dilation is located at F: (3,1)

The first step we must take is to find how many units point F is from vertex A.

In this case, F is 4 unit up and 8 units right of vertex A. (Reference the first image below to see how I found this)

Now we must multiply these dimensions (4 units and 8 units) by the factor of dilation (0.25)

4 * 0.25 = 1

8 * 0.25 = 2

This shows us how many dimensions away point [tex]A^{1}[/tex] is from point F:

1 unit vertically

2 units horizontally

Now how do we know if [tex]A^{1}[/tex] is above or below, left or right from point F? Easy! There has been no translations so the directions will be the same as the original:

Point F: (3, 1) ---> (3-2, 1-1) ---> (1, 0)

Reference the second image to see this

Hopefully this helps and makes sense. Let me know if you need further explanation or clarification.

~Just a girl in love with Shawn Mendes

Answer:

D 1,0

Step-by-step explanation:

credit to the one up above

Pam works on
commission from sellingE
stocks for a local bank.
She had a great month
and sold $2.5 million
dollars in investments
this month. She earns
4% commission on the
10% fee the bank
charges for sales over
$800,000. How much
did she earn?

Answers

Answer:

$10,000

Step by step:

Banks earnings(10%)=x

Pams earnings(4% of banks earnings)=y

10% of $2.5m= $250,000=x

4%(1/25) of $250,000=$10,000=y

theirs 2 questions please help?

Answers

Answer:

"past" is one

Step-by-step explanation:

Answer:

past is one okkkk kkkkjjjjj

Last year, Deanna bought 225 lamps when customers needed 250 lamps. This year, she bought 300 lamps but only sold 200 lamps. Which set of statements about the percent errors for the two years is true?
The percent error for this year was lower. Deanna estimated better last year.
The percent error for this year was higher. Deanna estimated better last year.
The percent error for this year was lower. Deanna estimated better this year.
The percent error for this year was higher. Deanna estimated better this year.

Answers

Answer:

The percent error for this year was higher. Deanna estimated better last year.

Step-by-step explanation:

Answer:

Step-by-step explanation:

6jkyuilyi

Jason and Henry go to the movie theater and purchase refreshments for their friends. Jason spends a total of $66.75 on 12 drinks and 1 bag of popcorn. Henry spends a total of $82.50 on 3 drinks and 10 bags of popcorn. Write a system of equations that can be used to find the price of one drink and the price of one bag of popcorn. Using these equations, determine and state the price of popcorn, to the nearest cent.

Answers

Answer:

The cost of one drink is $5

Step-by-step explanation:

3x-120x=82.50-667.5

120x-3x=667.50-82.50

117x=585

x=5

What is the difference between comprehensive and collision coverage in an auto policy​

Answers

Answer:

their both the same

Step-by-step explanation:

add and subtract divide multiply

36% of what number is 64.8?

Answers

Set up the equation 64.8 = .36x

Divide by .36 on both sides.

You get 180 = x.

36% of 180 is 64.8

Multiply .36 by 180 to double check.

Hope this helps

The answer is 180 hope this answer your problem

3. Olivia ordered 24 cupcakes and a layer cake. The layer cake cost $16 more than one
cupcake, and the total cost of the order was $53.50. What was the price of each cupcake?
a) Define a variable for the cost of one cupcake.
b) Write an expression to represent the cost of a layer cake.
c) Write and solve an equation.
d) Interpret your solution within the context of the problem.

Answers

Answer:

$2.14

Step-by-step explanation:

x =  price

24x+(x+16) = 53.5

25x+16 = 53.5

25x+53.5

x=53.5/25=2.14

PLEASE HELP
find value of x and y

Answers

Given :

There is a quadrilateral.Two sides of the quadrilateral are parallel .Four angles are 96° , 2x° , 94° & ( 3y + 44 )°.

To Find :-

The value of x & y .

Solution :-

Here , 96° & 2x° are co - interior angles and we know that the sum of co - Interior angles is 180°.

⇒ 96° + 2x = 180° .

⇒ 2x = 180° - 96° .

⇒ 2x = 84° .

⇒ x = [tex]\red{\sf \dfrac{84^{\circ}}{2}}[/tex]

[tex]\bf\longmapsto x = 42^{\circ}[/tex]

Hence value of x is 42° .

Similarly , 94° & ( 3y + 44) ° are co- interior angles

⇒ 94° + ( 3y + 44)° = 180° .

⇒ ( 3y + 44 )° = 180° - 94° .

⇒ 3y + 44° = 86°.

⇒ 3y = 86° - 44° .

⇒ 3y = 42° .

⇒ y = [tex]\sf \red{\dfrac{42^{\circ}}{3}}[/tex]

[tex]\bf\longmapsto y = 14^{\circ}[/tex]

Hence the value of y is 14°.

How many Significant Figures are in the number: 10.000?

Answers

Answer:

in 10.000, there are 5 significant figures.

hope this helps

3^100 dan 10^50 lebih besar mana? ​

Answers

Answer:

The bigger is 10 to the power of 50 or 10^50

Step-by-step explanation:

3 to the power of 100 = 3100 = 51537752 0732011331 0364611297 6562127270 2107522001. = place digits is 50

10 to the power of 50 = 1050 = 1 0000000000 0000000000 0000000000 0000000000 0000000000. = place digits is 51

12. What is a pronoun?

Answers

Answer:

Definition. A pronoun (I, me, he, she, herself, you, it, that, they, each, few, many, who, whoever, whose, someone, everybody, etc.) is a word that takes the place of a noun. In the sentence Joe saw Jill, and he waved at her, the pronouns he and her take the place of Joe and Jill, respectively.

(8.1 x 104)
+
(2.5 x 104). Express your answer in scientific notation.

Answers

Answer:

=10.6x104

Step-by-step explanation:

=8.1x104+2.5x104

=(8.1x104+2.5x104)

=10.6x104

(8.1 x104)+(2.5 x 104)

Multiply the numbers

842.4 + (2.5 x 104)

Multiply the numbers

842.4 +260

Now we do the same thing only different way

        2

       2.5

    x 104

________

           100

            000

             2 50 0

______________

            260

Multiply the digits

4 x 5 =20

Take the last digit and write it in the answer line

Carry the next digit and save it for the next calculation

Multiply the digits

4 x 2=8

Add the carried digit for the previous result

8+2 =10

Write the result in the answer line

Move the next digit and write it the next result below

Every digit multiplied by zero equals zero

Move to the next digit an write the next result below

Every digit multiplied by one stays the same

Add the numbers

Move the decimal sign as many times as there are decimal places in both factors combined.

Remove the unnecessary zeros

The result is 260

842.4+260

Add the numbers

1102.4

  842.4

+   260.0

_________

   1102.4

Write zeros to get the same number of decimal places.

Add the digits in the column

4+0=4

Write the result under the column

Add the digits in the column

2+0=2

Solution: 1102 .4

Alternate form : 5512/5 , 1102 2/5

PLEASE MARK ME AS BRAINLIEST

Find the area of the rectangle. The length is 1.25 and the width is 0.8 inches

Answers

The area is 1. Hope this helps.:)

Choose the definition, theorem, or postulate that justifies the statement.

A. Definition of a Right Angle
B. Definition of Complementary Angles
C. Congruent Complements Theorem
D. Complement Theorem
E. Definition of Supplementary Angles
F. Definition of Angle Bisector
G. Supplement Theorem
H. Definition of Perpendicular
I. Angle Addition Postulate
J. Vertical Angles Theorem
K. Congruent Supplements Theorem
L. Definition of Congruence

Answers

Answer:

E. Definition of Supplementary Angles

Step-by-step explanation:

A-55
B-65
C-75
D-90
Please help meee it’s a test

Answers

The answer would be B.

angle 2 = 90

180 (total degrees in a triangle) - 90 - 25 = 65

The answer is b the angle is 65

Charlene Langhorst's gross weekly salary is $680. Her federal withholding is $101.62. Her employer takes out the standard rates for FICA (Social Security/Medicare). The state tax is 1.5% of gross pay. Each week she pays $32.40 for medical insurance. What are Charlene's total weekly deductions? *

Answers

Answer:

Total deductions =  $196.24

Step-by-step explanation:

Gross weekly salary = $680

federal holding = $101.62

FICA standard rate = 7.65% of gross earnings

= 7.65% of $680 = 0.0765 × 680 = $52.02

∴ FICA deductions = $52.02

state tax = 1.5% of gross pay = 1.5% of $680

state tax = 0.015 × 680 = $10.2

∴ state tax = $10.2

Medical insurance = $32.40

Total Deductions = 101.62 + 52.02 + 10.2 + 32.40 = $196.24

I WILL GIVE BRAINLIEST! Which table of values corresponds to the graph below?

Answers

Answer:

the very bottom graph

Step-by-step explanation:

all of the points listed on the table match the graph

x values is found on the x axis (left to right)

y values are found on the y axis (up and down)

Answer:

see below

Step-by-step explanation:

From graph, points (2, 0) and (0, -2).

slope m = (y₂ - y₁) / (x₂ - x₁)

              = (-2 - 0) / (0 - 2)

              = -2 / -2

              = 1

y-intercept using anyone of the above points and m:

y = mx + b

0 = 1(2) + b

0 = 2 + b

b = -2

Equation of line using m and b:

y = 1x - 2

Use y = 1x - 2 to check the points given in tables:

All points in below table seems to work.

(0, -2)    

-2 = 0 - 2

-2 = -2

(1, -1)

-1 = 1 - 2

-1 = -1

(2, 0)

0 = 2 - 2

0 = 0

(4,2)

2 = 4 - 2

2 = 2

On a school fieldtrip with 344 students, they filled 5 buses, but 34
students couldn't fit on the buses and had to get rides with their parents.
How many students fit on each bus? *

Answers

Subtract 34 from 344 and then divide the answer by 5 so
344-34=310
310/5=62

A department store purchases screen-printed t-shirts at a cost of $5 per shirt. They mark up the price 150% (making the selling price 250% of the store's purchase price) and put them on the sales floor. Every month that a t-shirt doesn't sell, the store reduces the selling price by 25%.Which expression shows the selling price after one monthly price reduction?

Answers

Answer:

Kindly check explanation

Step-by-step explanation:

Given that:

Cost price = $5 per T - shirt

Markup = 150% or cost price

Every month which shirt doesn't sell, selling price drops by 25%

Selling price of Tshirt = (150% * $5) = $7.5

Price after 1 month = (100% - 25%) * $7.5 = $5.625

Hence, the equation could be expressed as :

Price after one month:

(1.5 * $5) - 0.75 * (1.5 * $5)

7.5 - 0.75(7.5)

Answer:

($5)(2.50) – ($5)(2.50)(0.25)

45 + 15(n − 1) for n = 6

Answers

Answer:

120

Step-by-step explanation:

Answer:

120

Step-by-step explanation:

Plug in what you know so: 45+15(6-1)

So, following the order of operations, you do the parentheses first so: 45+15(5)

Then, still following the order of operations, you'd multiply from left to right. So: 45+75

Then, you add from left to right, so you get 45+75=120

Your answer is 120.

is 10/2 an irrational number?

Answers

Answer:

yes it is because it is expressed in the form of a p/q

Step-by-step explanation:

If a number is expressed in the form of p/q then it is a rational number. Here p and q are integers, and q is not equal to 0. A rational number should have a numerator and denominator. Examples: 10/2, 30/3, 100/5.

so yes 10/2 is a rational number

oh and i mean yes its rational no its not irrational :)

No, it is not an irrational number.

Irrational numbers are those who cannot be turned into a fraction.

10/2 is a fraction. Therefore, it is a Rational number.

[tex]Edit:\;Also,\;if\;you\;were\;to\;make\;it\;a\;whole\;number\;then\;it\;would\;be\:5.[/tex] [tex]Which\;can\:be\;turned\;into\;5/1.[/tex]

To do so, divide the numerator (Top number) by the denominator (Bottom number).  

At Greenwood Landscaping, rock costs $47.50 per ton. How much would 2.5 tons cost?

$94.25
$118.75
$107.75
$332.50

Answers

Answer:

Hey!

Your answer is $118.75!

Step-by-step explanation: How did I get this?

STEP 1: So we know that for a ton of rock costs us $47.50...

To get 2 tons all we have to do is to MULTIPLY $47.50 by 2...

(47.50 x 2 = 95)

STEP 2: So for $95, you will get 2 tons of rock!

Now to get the .5 tons of rock in price form, we have to get the price of the whole 2.5 tons and DIVIDE BY 2

(47.50 ÷ 2 = 23.75)

STEP 3: So now we have the costs for 2 tons and the price for HALF a ton.

What we do now is easy...we ADD THE VALUES TOGTHER!

(95 + 23.75 = $118.75)

The result of $118.75 after adding them together is the PRICE OF 2 TONS!

I HOPE THIS HELPED YOU!

Answer:

$118.75

Step-by-step explanation:

Okay, we know a rock per ton is $47.50.

What we need to know is how much 2.5 tons cost.

So if we take our 2.5 tons and we multiply it with our $47.50, this is what it will look like:

($47.50 * 2.5) = $118.75

So 2.5 tons of rock costs $118.75

Have a great day! :)

Which of the following statements contain a variable?


A. The time it takes to wash the dishes plus 30 minutes.

B. The age of my friend.

C. The number of kittens in a litter.

D. There are 24 hours in a day. ​

Answers

Answer:The time it takes to wash the dishes plus 30 minutes

If y = -5 when x = 2, find y when x = 8. *

Answers

Answer:

y=1

Step-by-step explanation:

you add how many you went to one side

k/4 + 3 = 14

k= ?


k equals what

Answers

Answer:

Step-by-step explanation:

[tex]\frac{k}{4} + 3 = 14[/tex]

Subtract 3 from both sides

[tex]\frac{k}{4} = 14 - 3 \\\frac{k}{4} = 11[/tex]

Multiply both sides by 4 to find k.

k = 11 x 4 = 44

Other Questions
Car a travels 15 miles while car b travels 26 miles what is the ratio of car a to car b Which set contains the value below?50A.irrational B.Natural C. rational D.Integers Please help me correct these answers what is the most frequently occurring value in a set True Or false?The American Indian population increased during de sotos expedition. Consider a 5.430 g mixture of FeO and Fe3O4. You react this mixture with excess of oxygen to form 5.779 g Fe2O3. Calculate the percent mass of FeO in the original mixture. que son unos hiatos con vocales abierta? When you sit in a chair, your body exerts a downward force on the chair. Whatis the reaction force?A. The floor pushes up on the chair.B. The chair pushes down on the floor.C. Earth's gravity pulls up on the chainD. The chair pushes up on your body. how do i find the values for mean and standard deviation what is the common difference between successive terms in the sequence Figure out what x and y are. Please explain well! Please help me out with this! *It's just an extra practice assignment :) Question 320 ptsThe Indians won 14 out their last 20 games. What percent of their gamesdid they win? When vinegar and baking soda are combined, bubbles and foam are produced.What evidence demonstrates that a chemical change has occurred?F A color change occurs.G A gas is formed.H A solid is dissolved.J A precipitate is formed. insert two arithmetic means between 15 and 36 and they lived happily ever after is what figurative language Standard of living is the level at which Which equation represents the graph shown? A. y+7=-3(x-4)B. y+1=-3(x+2)C. y-4=3(x+7)D. y-2=3(x-1) how to create a code in pyton to ask the user to enter the number of switches and calculate the possible outcomes that can come from those many numbers of switches(on/off). Please help me with this What is the range of the functiongraphed below?(3,3)